Difference between revisions of "1996 AIME Problems/Problem 15"
m (→Solution 1: forgot <asy>) |
(easier) |
||
Line 4: | Line 4: | ||
__TOC__ | __TOC__ | ||
== Solution == | == Solution == | ||
− | === Solution 1 === | + | === Solution 1 (trignometry) === |
<center><asy>size(180); pathpen = black+linewidth(0.7); | <center><asy>size(180); pathpen = black+linewidth(0.7); | ||
pair B=(0,0), A=expi(pi/4), C=IP(A--A + 2*expi(17*pi/12), B--(3,0)), D=A+C, O=IP(A--C,B--D); | pair B=(0,0), A=expi(pi/4), C=IP(A--A + 2*expi(17*pi/12), B--(3,0)), D=A+C, O=IP(A--C,B--D); | ||
Line 29: | Line 29: | ||
The only value of <math>\theta</math> that fits in this context comes from <math>4 \cos^2 2\theta - 3 = 0 \Longrightarrow \cos 2\theta = \frac{\sqrt{3}}{2} \Longrightarrow \theta = 15^{\circ}</math>. The answer is <math>\lfloor 1000r \rfloor = \left\lfloor 1000 \cdot \frac{180 - 5\theta}{180 - 3\theta} \right\rfloor = \left \lfloor \frac{7000}{9} \right \rfloor = \boxed{777}</math>. | The only value of <math>\theta</math> that fits in this context comes from <math>4 \cos^2 2\theta - 3 = 0 \Longrightarrow \cos 2\theta = \frac{\sqrt{3}}{2} \Longrightarrow \theta = 15^{\circ}</math>. The answer is <math>\lfloor 1000r \rfloor = \left\lfloor 1000 \cdot \frac{180 - 5\theta}{180 - 3\theta} \right\rfloor = \left \lfloor \frac{7000}{9} \right \rfloor = \boxed{777}</math>. | ||
− | === Solution 2 === <!-- I would put this solution first, but needs a bit of formatting for clarity - azjps --> | + | === Solution 2 (trignometry) === |
+ | Define <math>\theta</math> as above. Since <math>\angle CAB = \angle CBO</math>, it follows that <math>\triangle COB \sim \triangle CBA</math>, and so <math>\frac{CO}{BC} = \frac{BC}{AC} \Longrightarrow BC^2 = AC \cdot CO = 2CO^2 \Longrightarrow BC = CO\sqrt{2}</math>. The [[Law of Sines]] on <math>\triangle BOC</math> yields that | ||
+ | |||
+ | <cmath>\frac{BC}{CO} = \frac{\sin (180-3\theta)}{\sin 2\theta} = \frac{\sin 3\theta}{\sin 2\theta} = \sqrt{2}</cmath> | ||
+ | |||
+ | Expanding using the sine double and triple angle formulas, we have | ||
+ | |||
+ | <cmath>2\sqrt {2} \sin \theta \cos \theta = \sin\theta( - 4\sin ^2 \theta + 3) \Longrightarrow \sin \left\theta (4\cos^2\theta - 2\sqrt {2} \cos \theta - 1\right) = 0.</cmath> | ||
+ | |||
+ | By the [[quadratic formula]], we have <math>\cos \theta = \frac {2\sqrt {2} \pm \sqrt {8 + 4 \cdot 1 \cdot 4}}{8} = \frac {\sqrt {6} \pm \sqrt {2}}{4}</math>, so <math>\theta = 15^{\circ}</math> (as the other roots are too large to make sense in context). The answer follows as above. | ||
+ | |||
+ | === Solution 3 === <!-- I would put this solution first, but needs a bit of formatting for clarity - azjps --> | ||
We will focus on <math>\triangle ABC</math>. Let <math>\angle ABO = x</math>, so <math>\angle BAO = \angle OBC = 2x</math>. Draw the [[perpendicular]] from <math>C</math> intersecting <math>AB</math> at <math>H</math>. [[Without loss of generality]], let <math>AO = CO = 1</math>. Then <math>HO = 1</math>, since <math>O</math> is the [[circumcenter]] of <math>\triangle AHC</math>. Then <math>\angle OHA = 2x</math>. | We will focus on <math>\triangle ABC</math>. Let <math>\angle ABO = x</math>, so <math>\angle BAO = \angle OBC = 2x</math>. Draw the [[perpendicular]] from <math>C</math> intersecting <math>AB</math> at <math>H</math>. [[Without loss of generality]], let <math>AO = CO = 1</math>. Then <math>HO = 1</math>, since <math>O</math> is the [[circumcenter]] of <math>\triangle AHC</math>. Then <math>\angle OHA = 2x</math>. | ||
Line 36: | Line 47: | ||
<center><math>\frac {CO}{BC} = \frac {BC}{AC} \implies 2 = BC^2 = \implies BC = \sqrt {2}. </math></center> | <center><math>\frac {CO}{BC} = \frac {BC}{AC} \implies 2 = BC^2 = \implies BC = \sqrt {2}. </math></center> | ||
− | Then by the [[Pythagorean Theorem]], <math>1^2 + HC^2 = \left(\sqrt {2}\right)^2\implies HC = 1</math>. That makes <math>\triangle HOC</math> equilateral. Then <math>\angle HOC = 4x = 60 \implies x = 15</math>. | + | Then by the [[Pythagorean Theorem]], <math>1^2 + HC^2 = \left(\sqrt {2}\right)^2\implies HC = 1</math>. That makes <math>\triangle HOC</math> equilateral. Then <math>\angle HOC = 4x = 60 \implies x = 15</math>. The answer follows as above. |
− | |||
− | |||
== See also == | == See also == |
Revision as of 17:06, 27 August 2008
Problem
In parallelogram , let be the intersection of diagonals and . Angles and are each twice as large as angle , and angle is times as large as angle . Find the greatest integer that does not exceed .
Contents
Solution
Solution 1 (trignometry)
Let . Then , , and . Since is a parallelogram, it follows that . By the Law of Sines on ,
Dividing the two equalities yields
Pythagorean and product-to-sum identities yield
and the double and triple angle () formulas further simplify this to
The only value of that fits in this context comes from . The answer is .
Solution 2 (trignometry)
Define as above. Since , it follows that , and so . The Law of Sines on yields that
Expanding using the sine double and triple angle formulas, we have
\[2\sqrt {2} \sin \theta \cos \theta = \sin\theta( - 4\sin ^2 \theta + 3) \Longrightarrow \sin \left\theta (4\cos^2\theta - 2\sqrt {2} \cos \theta - 1\right) = 0.\] (Error compiling LaTeX. Unknown error_msg)
By the quadratic formula, we have , so (as the other roots are too large to make sense in context). The answer follows as above.
Solution 3
We will focus on . Let , so . Draw the perpendicular from intersecting at . Without loss of generality, let . Then , since is the circumcenter of . Then .
By the Exterior Angle Theorem, and . That implies that . That makes . Then since by AA ( and reflexive on ), .
Then by the Pythagorean Theorem, . That makes equilateral. Then . The answer follows as above.
See also
1996 AIME (Problems • Answer Key • Resources) | ||
Preceded by Problem 14 |
Followed by Final Problem | |
1 • 2 • 3 • 4 • 5 • 6 • 7 • 8 • 9 • 10 • 11 • 12 • 13 • 14 • 15 | ||
All AIME Problems and Solutions |